Using the limit comparison test to prove conv or div

In summary, the limit comparison test says that if you have two positive series, sum An and sum Bn, let C=lim n to infinity of An/Bn and (i) if 0< C < infinity; then both series converge or both diverge; (ii) if C = 0 and sum Bn converges, so does sum An; (iii) if C = infinity and sum Bn diverges, so does sum An.
  • #1
Randall
22
0

Homework Statement


Use the limit comparison test to prove convergence or divergence for the series sum from n=1 to infinity for ((5n^3)+1)/((2^n)((n^3)+n+1))

Homework Equations


The limit comparison test says that if you have two positive series, sum An and sum Bn, let C=lim n to infinity of An/Bn and (i) if 0< C < infinity; then both series converge or both diverge; (ii) if C = 0 and sum Bn converges, so does sum An; (iii) if C = infinity and sum Bn diverges, so does sum An.

The Attempt at a Solution


For An, I used L'Hospital's rule (twice) to get lim as n to infinity of 30/infinity, which equals 0. For Bn, I chose Bn = 2^n, which is a divergent geometric series with an r of 2 (>1). This didn't help me apply the limit comparison test, so I changed my Bn to the convergent geometric series 0.5^n, which has an r of 0.5 (<1). This also didn't help me apply the limit comparison test. Can someone help me please? Thanks so much in advance.
 
Physics news on Phys.org
  • #2
You don't take the the limits of ##A_n## and ##B_n## separately for the limit comparison test. What you want to try for##B_n## is not ##2^n## but ##\frac 1 {2^n}## which you have in your series. So you should be calculating the limit of $$
\frac{\frac{5n^3+1}{(2^n)(n^3+n+1)}}{\frac 1 {2^n} }$$Simplify that and work out that limit. You shouldn't need L'Hospital's rule to work that limit.
 
  • #3
Randall said:
so I changed my Bn to the convergent geometric series 0.5^n, which has an r of 0.5 (<1). This also didn't help me apply the limit comparison test. .

Show why didn't it work to use [itex]B_n = 0.5^n = \frac{1}{2^n} [/itex]
 
  • #4
Hi Stephen: so when I used Bn = 0.5^n, and then analyzed it as n goes to infinity, I got Bn = 0. And, since I already determined that An=0, that makes my C = 0/0, which is undefined, yes? So I can't apply the limit comparison test for Bn being a convergent series.
 
  • #5
Randall said:
Hi Stephen: so when I used Bn = 0.5^n, and then analyzed it as n goes to infinity, I got Bn = 0. And, since I already determined that An=0, that makes my C = 0/0, which is undefined, yes? So I can't apply the limit comparison test for Bn being a convergent series.
Did you read the first sentence of my post #2? Or post #2 at all?
 
  • #6
Randall said:
Hi Stephen: so when I used Bn = 0.5^n, and then analyzed it as n goes to infinity, I got Bn = 0. And, since I already determined that An=0, that makes my C = 0/0, which is undefined, yes? .

No. If that were true the l'Hosptials rule wouldn't work. Reduce the fraction [itex]\frac{A_n}{B_n} [/itex] and take the limit of [itex] \frac{A_n}{B_n} [/itex] using l'Hospital's rule.
 
  • #7
LCKurtz: Ok thanks I tried that but still had to use L'Hospital's rule to evaluate the limit as n goes to infinity. For Bn, I ended up with lim n to inf of 15 / 6n = 15/infinity = 0. So, since An = 0, I get a C of 0/0, which is undefined yes, or is C = 0? Thanks.
 
  • #8
LCKurtz, no I had not read it when I was responding to Stephen, but I have read it since and have replied to you.
 
  • #9
You need to show your work. Show us what you took the limit of and how you did it.
 
  • #10
Randall said:
LCKurtz: Ok thanks I tried that but still had to use L'Hospital's rule to evaluate the limit as n goes to infinity. For Bn, I ended up with lim n to inf of 15 / 6n = 15/infinity = 0. So, since An = 0, I get a C of 0/0, which is undefined yes, or is C = 0? Thanks.
What LCKurtz meant in the first sentence of post 2 is that
$$C = \lim_{n \to \infty} \frac{A_n}{B_n} \ne \frac {\lim_{n \to \infty} A_n}{\lim_{n \to \infty} B_n}.$$ From what you've written, it sounds like you're finding the quotient of the two limits, which isn't what you should be doing. You should be calculating the limit of the quotient ##A_n/B_n##.
 
  • #11
To your quoted formulation of the comparison test I prefer the one in the blue box here http://tutorial.math.lamar.edu/Classes/CalcII/SeriesCompTest.aspx or the one in the first lines here http://tutorial.math.lamar.edu/Classes/CalcII/SeriesCompTest.aspx .

Because you are not asked to find a limit but only establish that there is one, the comparison test allows you to work with < instead of = , which allows you considerable freedom in eliminating awkward terms you don't want and replacing your ((5n^3)+1)/((2^n)((n^3)+n+1))
with an easier and more obviously convergent expression.
 

What is the limit comparison test?

The limit comparison test is a method used in mathematics to determine the convergence or divergence of a series. It involves comparing the given series to a known series with known convergence or divergence.

When should I use the limit comparison test?

The limit comparison test should be used when the series in question is difficult to evaluate directly, or when other tests, such as the ratio or root test, are inconclusive.

What is the process for using the limit comparison test?

To use the limit comparison test, first identify the given series and a known series with known convergence or divergence. Then, take the limit of the ratio of the two series. If the limit is a positive number, the two series have the same convergence or divergence. If the limit is zero or infinite, the two series have opposite convergence or divergence.

Can the limit comparison test be used for both infinite and finite series?

Yes, the limit comparison test can be used for both infinite and finite series. However, it is most commonly used for infinite series, as it is more difficult to determine the convergence or divergence of these types of series.

What is the benefit of using the limit comparison test?

The limit comparison test is beneficial because it is often easier to use and more conclusive than other tests, such as the ratio or root test. It also allows for the comparison of series with different terms, making it a versatile tool for determining convergence or divergence.

Similar threads

  • Calculus and Beyond Homework Help
Replies
1
Views
1K
  • Calculus and Beyond Homework Help
Replies
29
Views
1K
  • Calculus and Beyond Homework Help
Replies
2
Views
196
  • Calculus and Beyond Homework Help
Replies
2
Views
995
  • Calculus and Beyond Homework Help
Replies
6
Views
904
  • Calculus and Beyond Homework Help
Replies
2
Views
714
  • Calculus and Beyond Homework Help
Replies
1
Views
268
  • Calculus and Beyond Homework Help
Replies
14
Views
1K
  • Calculus and Beyond Homework Help
Replies
2
Views
846
  • Calculus and Beyond Homework Help
Replies
4
Views
1K
Back
Top